Submitted by devansh shringi

Algebra Level 4

A cubic polynomial

f ( x ) = 8 x 3 3 x 2 3 x 1 f(x) = 8x^3-3x^2-3x-1

has one real root of the form a 3 + b 3 + 1 c \frac{ \sqrt[3]{a} + \sqrt[3]{b} + 1} { c} , where a , b , c a, b, c are positive integers. Find a + b + c a + b + c .


The answer is 98.

This section requires Javascript.
You are seeing this because something didn't load right. We suggest you, (a) try refreshing the page, (b) enabling javascript if it is disabled on your browser and, finally, (c) loading the non-javascript version of this page . We're sorry about the hassle.

5 solutions

Daniel Liu
Dec 20, 2013

We have that 9 x 3 = ( x + 1 ) 3 9x^3 = (x+1)^3 , so it follows that 9 3 x = x + 1 \sqrt[3]{9}x = x+1 . Solving for x x yields 1 9 3 1 = 81 3 + 9 3 + 1 8 \frac{1}{\sqrt[3]{9}-1} = \frac{\sqrt[3]{81}+\sqrt[3]{9}+1}{8} , so the answer is 098 \boxed{098} .

This user blatantly stole the problem from the 2013 AIME I problem 5. It seems fitting that I should, similarly, blatantly copy a solution that can be found here: http://www.artofproblemsolving.com/Wiki/index.php/2013 AIME I Problems/Problem 5

@devansh shringi: It is OK for you to get inspiration from other problems, but it is not OK to just copy them and submit them as your own.

Yes, thank you, Daniel!

It is OK to share nice problems you have come across, provided that the solution is not obviously available on the Internet, and that you give proper credit. In particular, if you want to share an AIME problem, modify it before submitting and indicate that your problem is based on an AIME problem. You may also want to reveal the exact source afterwards. The recent AIME problems should probably be avoided altogether unless they can be SUBSTANTIALLY modified.

Alexander Borisov - 7 years, 5 months ago

Oh man! I do not these stealings!

User "Bruce Wayne" has done the same thing yesterday with three problems from Nick's Mathematical Puzzles.

We should be able to "flag" these problems.

Guilherme Dela Corte - 7 years, 5 months ago

Wow this was a difficult problem. But how on earth did you rationalise that denominator?

Nahom Yemane - 7 years, 5 months ago

Log in to reply

The factorization that ( a b ) ( a 2 + a b + b 2 ) = a 3 b 3 (a - b)(a^2 + ab + b^2) = a^3 - b^3 .

Michael Tong - 7 years, 5 months ago

Yea, I knew that I saw this problem before.

Michael Tong - 7 years, 5 months ago

Awesome! Daniel

jinay patel - 7 years, 5 months ago

I really liked the simplicity of this (apparently hard) problem.

Let's rewrite f ( x ) = 8 x 3 3 x 2 3 x 1 f(x) = 8x^3 - 3x^2 - 3x - 1 as f ( x ) = 9 x 3 ( x + 1 ) 3 f(x) = 9x^3 - (x+1)^3 .

Knowing ( a b ) ( a 2 + a b + b 2 ) = a 3 b 3 (a - b)(a^2 + ab + b^2)= a^3 - b^3 , we can simplify f ( x ) f(x) to ( 9 3 x x 1 ) ( 81 3 + 9 3 ( x 1 ) + ( x 1 ) 2 ) = 0 (\sqrt[3]{9}x - x - 1)(\sqrt[3]{81} +\sqrt[3]{9}(x-1) + (x-1)^2) = 0 .

It is clear that the expression 9 3 x x 1 = 0 \sqrt[3]{9}x - x - 1 = 0 holds the real root. Thus x = 1 9 3 1 x = \dfrac{1}{\sqrt[3]{9} - 1} .

Remembering ( a 1 ) ( a 2 + a + 1 ) = x 3 1 (a - 1)(a^2 + a + 1)= x^3 - 1 , we can use this trick to simplify x x 's fraction to x = 81 3 + 9 3 + 1 8 x = \dfrac { \sqrt[3]{81} + \sqrt[3]{9} + 1 }{8} . Thus a + b + c = 98 a + b + c = 98 .

Daniel Thompson
Dec 20, 2013

Let y = x 1 8 y=x-\frac{1}{8} , then 8 x 3 3 x 2 3 x 1 = 0 x 3 3 8 x 2 3 8 x 1 8 = 0 y 3 27 64 y 90 512 = 0 8x^3-3x^2-3x-1=0 \iff x^3-\frac{3}{8}x^2-\frac{3}{8}x-\frac{1}{8}=0 \iff y^3-\frac{27}{64}y-\frac{90}{512}=0 . Now if y = u + v y=u+v then y 3 3 u v y ( u 3 + v 3 ) = 0 y^3-3uvy-(u^3+v^3)=0 has root y = u + v y=u+v . I need 9 64 = u v \frac{9}{64}=uv and 90 512 = u 3 + v 3 \frac{90}{512}=u^3+v^3 . From these two equations, u 6 90 512 u 3 + 9 3 6 4 3 = 0 u^6-\frac{90}{512}u^3+\frac{9^3}{64^3}=0 so u 3 = 1 2 ( 90 512 + 9 0 2 51 2 2 4 9 3 6 4 3 ) u^3=\frac{1}{2} \left(\frac{90}{512}+\sqrt{\frac{90^2}{512^2}-4\frac{9^3}{64^3}} \right ) and v 3 = 1 2 ( 90 512 9 0 2 51 2 2 4 9 3 6 4 3 ) v^3=\frac{1}{2} \left (\frac{90}{512}-\sqrt{\frac{90^2}{512^2}-4\frac{9^3}{64^3}} \right ) Therefore y = 1 2 ( 90 512 + 9 2 1 0 2 51 2 2 4 9 3 6 4 3 ) 3 + 1 2 ( 90 512 9 2 1 0 2 51 2 2 4 9 3 6 4 3 ) 3 y=\sqrt[3]{\frac{1}{2} \left ( \frac{90}{512}+\sqrt{\frac{9^2 \cdot 10^2}{512^2}-4\frac{9^3}{64^3}}\right )}+\sqrt[3]{\frac{1}{2} \left (\frac{90}{512}-\sqrt{\frac{9^2 \cdot 10^2}{512^2}-4\frac{9^3}{64^3}} \right )} y = 1 2 10 ( 90 + 9 2 ( 100 4 9 ) ) 3 + 1 2 10 ( 90 9 2 ( 100 4 9 ) ) 3 y=\sqrt[3]{\frac{1}{2^{10}} \left (90+\sqrt{9^2(100-4 \cdot 9)} \right )}+\sqrt[3]{\frac{1}{2^{10}} \left (90-\sqrt{9^2(100-4 \cdot 9)}\right ) } y = 1 2 10 ( 90 + 72 ) 3 + 1 2 10 ( 90 72 ) 3 y=\sqrt[3]{\frac{1}{2^{10}}(90+72)}+\sqrt[3]{\frac{1}{2^{10}}(90-72)} y = 3 + 3 2 3 8 x = 8 1 1 3 + 9 1 3 + 1 8 y=\frac{3+3^{\frac{2}{3}}}{8} \iff x=\frac{81^{\frac{1}{3}}+9^{\frac{1}{3}}+1}{8} a + b + c = 98 a+b+c=98

Quite the bulldozer you've used to dig a 3 inch hole in soil...

Daniel Liu - 7 years, 5 months ago

Log in to reply

Yes, very inelegant and time consuming. I regret posting this after seeing the simplicity of the other solutions.

Daniel Thompson - 7 years, 5 months ago

Log in to reply

Even though it is not a good way, but it is still one of the way to approach it

敬全 钟 - 7 years, 5 months ago
Mohit Sharma
Dec 29, 2013

By applying cardano's method ,we get the real root as: x = 3 4 3 + 3 2 3 + 1 8 x = \frac{3^{\frac{4}{3}}+3^{\frac{2}{3}}+1}{8} Hence, a = 81, b = 9, c = 8 a + b + c = 81 + 9 + 8 = 98 \Rightarrow a+b+c = 81 + 9 + 8 = \boxed{98}

Roger Jin
Dec 21, 2013

f(x) = 0 adding x cubed to both sides yields 9x^3 = x^3 +3x^2 +3x +1 = (x+1)^3 taking cube root of both sides and then solving for x and rationalizing the denominator, x = (cubert (81) + cubert (9) + 1)/8 so the answer is 98

0 pending reports

×

Problem Loading...

Note Loading...

Set Loading...